Gravity magic - How does it work?

The name of the pictureThe name of the pictureThe name of the pictureClash Royale CLAN TAG#URR8PPP












21












$begingroup$


This question graduated from the Sandbox for Proposed Questions




Gravity magic - How does it work?



(What physical law can magic manipulate in order to affect gravity?)



In my world everyone is capable of casting magic. It is possible for a single person to cast as many different types of magic as he/she wishes, assuming they know the spells. Every person also has an affinity to a specific type/form of magic, e.g. projectile magic, water magic or defensive magic. One of the main characters in my story has an affinity to gravity-based spells, so it is much easier and more intuitive to use them for her.



What I'm interested in is not how magic works in this context (assume there is some amount of recharging MP within every person that is used to cast spells), but rather how the gravitational effects caused by the magic could be explained.



I'm looking for a "generic" solution that explains explicitly this type of magic, not every single spell/usage, but rather "gravity-type magic" itself. I don't intend to explain my readers every single law of physics, space-time etc. in every detail, but the main aspect of "how" should come across.



When answering, take the following scenarios as use-cases for the magic to check:



  • Slowing down yourself down when falling, so you don't hit the ground at full speed (as near to 0m/s as possible, at the very least you shouldn't be injured by landing) (e.g. jumping from higher up)

  • Levitating yourself / other objects, so they don't move in any direction

    • Bonus points: still won't move if influenced by another force (e.g. a thrown stone)


  • Pushing objects out of your way (e.g. a locked door)

  • Ability to combine multiple "gravity fields", so you could simultaneously keep a cup in place and rotate it upwards-down, while still keeping the liquid inside

I've also found the question How do I grant superheroes gravitational manipulation, justified scientifically?, but it is somewhat different than my question. My question focuses on how to actually perform the gravity-based magic and not on how you could justify the existence of such powers.




Edit:
Thank you for all the great answers! I'll go with the Answer by Chronocidal, since it fits my own idea of how it should work best of the existing answers.










share|improve this question











$endgroup$







  • 8




    $begingroup$
    Good job using the sandbox.
    $endgroup$
    – L.Dutch
    Mar 15 at 7:14






  • 1




    $begingroup$
    Do note that changing your mass doesn't make you fall any slower unless you also change your buoyancy in air, so magic is in the air! Allez, allez, allez...
    $endgroup$
    – user6760
    Mar 15 at 10:51







  • 3




    $begingroup$
    Brandon Sanderson's "The Stormlight Archive" series present a nice example of gravity magic. Aside from being a very good book, it may inspire you.
    $endgroup$
    – Echox
    Mar 15 at 14:30






  • 2




    $begingroup$
    @Echox Come for the gravity magic, stay for the bizarrely alien worldbuilding, the setting that manages to make huge fantasy swords and magic power armor actually work, the spren, Shadesmar, the secret societies, the Unmade, the dozens of POV characters, the political intrigue, Wit, the personal drama... take your pick. Stormlight has something for everyone!
    $endgroup$
    – Mason Wheeler
    Mar 15 at 16:22
















21












$begingroup$


This question graduated from the Sandbox for Proposed Questions




Gravity magic - How does it work?



(What physical law can magic manipulate in order to affect gravity?)



In my world everyone is capable of casting magic. It is possible for a single person to cast as many different types of magic as he/she wishes, assuming they know the spells. Every person also has an affinity to a specific type/form of magic, e.g. projectile magic, water magic or defensive magic. One of the main characters in my story has an affinity to gravity-based spells, so it is much easier and more intuitive to use them for her.



What I'm interested in is not how magic works in this context (assume there is some amount of recharging MP within every person that is used to cast spells), but rather how the gravitational effects caused by the magic could be explained.



I'm looking for a "generic" solution that explains explicitly this type of magic, not every single spell/usage, but rather "gravity-type magic" itself. I don't intend to explain my readers every single law of physics, space-time etc. in every detail, but the main aspect of "how" should come across.



When answering, take the following scenarios as use-cases for the magic to check:



  • Slowing down yourself down when falling, so you don't hit the ground at full speed (as near to 0m/s as possible, at the very least you shouldn't be injured by landing) (e.g. jumping from higher up)

  • Levitating yourself / other objects, so they don't move in any direction

    • Bonus points: still won't move if influenced by another force (e.g. a thrown stone)


  • Pushing objects out of your way (e.g. a locked door)

  • Ability to combine multiple "gravity fields", so you could simultaneously keep a cup in place and rotate it upwards-down, while still keeping the liquid inside

I've also found the question How do I grant superheroes gravitational manipulation, justified scientifically?, but it is somewhat different than my question. My question focuses on how to actually perform the gravity-based magic and not on how you could justify the existence of such powers.




Edit:
Thank you for all the great answers! I'll go with the Answer by Chronocidal, since it fits my own idea of how it should work best of the existing answers.










share|improve this question











$endgroup$







  • 8




    $begingroup$
    Good job using the sandbox.
    $endgroup$
    – L.Dutch
    Mar 15 at 7:14






  • 1




    $begingroup$
    Do note that changing your mass doesn't make you fall any slower unless you also change your buoyancy in air, so magic is in the air! Allez, allez, allez...
    $endgroup$
    – user6760
    Mar 15 at 10:51







  • 3




    $begingroup$
    Brandon Sanderson's "The Stormlight Archive" series present a nice example of gravity magic. Aside from being a very good book, it may inspire you.
    $endgroup$
    – Echox
    Mar 15 at 14:30






  • 2




    $begingroup$
    @Echox Come for the gravity magic, stay for the bizarrely alien worldbuilding, the setting that manages to make huge fantasy swords and magic power armor actually work, the spren, Shadesmar, the secret societies, the Unmade, the dozens of POV characters, the political intrigue, Wit, the personal drama... take your pick. Stormlight has something for everyone!
    $endgroup$
    – Mason Wheeler
    Mar 15 at 16:22














21












21








21


2



$begingroup$


This question graduated from the Sandbox for Proposed Questions




Gravity magic - How does it work?



(What physical law can magic manipulate in order to affect gravity?)



In my world everyone is capable of casting magic. It is possible for a single person to cast as many different types of magic as he/she wishes, assuming they know the spells. Every person also has an affinity to a specific type/form of magic, e.g. projectile magic, water magic or defensive magic. One of the main characters in my story has an affinity to gravity-based spells, so it is much easier and more intuitive to use them for her.



What I'm interested in is not how magic works in this context (assume there is some amount of recharging MP within every person that is used to cast spells), but rather how the gravitational effects caused by the magic could be explained.



I'm looking for a "generic" solution that explains explicitly this type of magic, not every single spell/usage, but rather "gravity-type magic" itself. I don't intend to explain my readers every single law of physics, space-time etc. in every detail, but the main aspect of "how" should come across.



When answering, take the following scenarios as use-cases for the magic to check:



  • Slowing down yourself down when falling, so you don't hit the ground at full speed (as near to 0m/s as possible, at the very least you shouldn't be injured by landing) (e.g. jumping from higher up)

  • Levitating yourself / other objects, so they don't move in any direction

    • Bonus points: still won't move if influenced by another force (e.g. a thrown stone)


  • Pushing objects out of your way (e.g. a locked door)

  • Ability to combine multiple "gravity fields", so you could simultaneously keep a cup in place and rotate it upwards-down, while still keeping the liquid inside

I've also found the question How do I grant superheroes gravitational manipulation, justified scientifically?, but it is somewhat different than my question. My question focuses on how to actually perform the gravity-based magic and not on how you could justify the existence of such powers.




Edit:
Thank you for all the great answers! I'll go with the Answer by Chronocidal, since it fits my own idea of how it should work best of the existing answers.










share|improve this question











$endgroup$




This question graduated from the Sandbox for Proposed Questions




Gravity magic - How does it work?



(What physical law can magic manipulate in order to affect gravity?)



In my world everyone is capable of casting magic. It is possible for a single person to cast as many different types of magic as he/she wishes, assuming they know the spells. Every person also has an affinity to a specific type/form of magic, e.g. projectile magic, water magic or defensive magic. One of the main characters in my story has an affinity to gravity-based spells, so it is much easier and more intuitive to use them for her.



What I'm interested in is not how magic works in this context (assume there is some amount of recharging MP within every person that is used to cast spells), but rather how the gravitational effects caused by the magic could be explained.



I'm looking for a "generic" solution that explains explicitly this type of magic, not every single spell/usage, but rather "gravity-type magic" itself. I don't intend to explain my readers every single law of physics, space-time etc. in every detail, but the main aspect of "how" should come across.



When answering, take the following scenarios as use-cases for the magic to check:



  • Slowing down yourself down when falling, so you don't hit the ground at full speed (as near to 0m/s as possible, at the very least you shouldn't be injured by landing) (e.g. jumping from higher up)

  • Levitating yourself / other objects, so they don't move in any direction

    • Bonus points: still won't move if influenced by another force (e.g. a thrown stone)


  • Pushing objects out of your way (e.g. a locked door)

  • Ability to combine multiple "gravity fields", so you could simultaneously keep a cup in place and rotate it upwards-down, while still keeping the liquid inside

I've also found the question How do I grant superheroes gravitational manipulation, justified scientifically?, but it is somewhat different than my question. My question focuses on how to actually perform the gravity-based magic and not on how you could justify the existence of such powers.




Edit:
Thank you for all the great answers! I'll go with the Answer by Chronocidal, since it fits my own idea of how it should work best of the existing answers.







science-based magic gravity






share|improve this question















share|improve this question













share|improve this question




share|improve this question








edited Mar 22 at 9:15







Tobias F.

















asked Mar 15 at 7:03









Tobias F.Tobias F.

288310




288310







  • 8




    $begingroup$
    Good job using the sandbox.
    $endgroup$
    – L.Dutch
    Mar 15 at 7:14






  • 1




    $begingroup$
    Do note that changing your mass doesn't make you fall any slower unless you also change your buoyancy in air, so magic is in the air! Allez, allez, allez...
    $endgroup$
    – user6760
    Mar 15 at 10:51







  • 3




    $begingroup$
    Brandon Sanderson's "The Stormlight Archive" series present a nice example of gravity magic. Aside from being a very good book, it may inspire you.
    $endgroup$
    – Echox
    Mar 15 at 14:30






  • 2




    $begingroup$
    @Echox Come for the gravity magic, stay for the bizarrely alien worldbuilding, the setting that manages to make huge fantasy swords and magic power armor actually work, the spren, Shadesmar, the secret societies, the Unmade, the dozens of POV characters, the political intrigue, Wit, the personal drama... take your pick. Stormlight has something for everyone!
    $endgroup$
    – Mason Wheeler
    Mar 15 at 16:22













  • 8




    $begingroup$
    Good job using the sandbox.
    $endgroup$
    – L.Dutch
    Mar 15 at 7:14






  • 1




    $begingroup$
    Do note that changing your mass doesn't make you fall any slower unless you also change your buoyancy in air, so magic is in the air! Allez, allez, allez...
    $endgroup$
    – user6760
    Mar 15 at 10:51







  • 3




    $begingroup$
    Brandon Sanderson's "The Stormlight Archive" series present a nice example of gravity magic. Aside from being a very good book, it may inspire you.
    $endgroup$
    – Echox
    Mar 15 at 14:30






  • 2




    $begingroup$
    @Echox Come for the gravity magic, stay for the bizarrely alien worldbuilding, the setting that manages to make huge fantasy swords and magic power armor actually work, the spren, Shadesmar, the secret societies, the Unmade, the dozens of POV characters, the political intrigue, Wit, the personal drama... take your pick. Stormlight has something for everyone!
    $endgroup$
    – Mason Wheeler
    Mar 15 at 16:22








8




8




$begingroup$
Good job using the sandbox.
$endgroup$
– L.Dutch
Mar 15 at 7:14




$begingroup$
Good job using the sandbox.
$endgroup$
– L.Dutch
Mar 15 at 7:14




1




1




$begingroup$
Do note that changing your mass doesn't make you fall any slower unless you also change your buoyancy in air, so magic is in the air! Allez, allez, allez...
$endgroup$
– user6760
Mar 15 at 10:51





$begingroup$
Do note that changing your mass doesn't make you fall any slower unless you also change your buoyancy in air, so magic is in the air! Allez, allez, allez...
$endgroup$
– user6760
Mar 15 at 10:51





3




3




$begingroup$
Brandon Sanderson's "The Stormlight Archive" series present a nice example of gravity magic. Aside from being a very good book, it may inspire you.
$endgroup$
– Echox
Mar 15 at 14:30




$begingroup$
Brandon Sanderson's "The Stormlight Archive" series present a nice example of gravity magic. Aside from being a very good book, it may inspire you.
$endgroup$
– Echox
Mar 15 at 14:30




2




2




$begingroup$
@Echox Come for the gravity magic, stay for the bizarrely alien worldbuilding, the setting that manages to make huge fantasy swords and magic power armor actually work, the spren, Shadesmar, the secret societies, the Unmade, the dozens of POV characters, the political intrigue, Wit, the personal drama... take your pick. Stormlight has something for everyone!
$endgroup$
– Mason Wheeler
Mar 15 at 16:22





$begingroup$
@Echox Come for the gravity magic, stay for the bizarrely alien worldbuilding, the setting that manages to make huge fantasy swords and magic power armor actually work, the spren, Shadesmar, the secret societies, the Unmade, the dozens of POV characters, the political intrigue, Wit, the personal drama... take your pick. Stormlight has something for everyone!
$endgroup$
– Mason Wheeler
Mar 15 at 16:22











9 Answers
9






active

oldest

votes


















14












$begingroup$


What I'm interested in is not how magic works in this context (assume there is some amount of recharging MP within every person that is used to cast spells), but rather how the gravitational effects caused by the magic could be explained.




If you stick to Einstein's model for gravity, you can explain your magic via the ability of manipulating the curvature of space-time.



The Gravitational force experienced by an object (the Einstein tensor) subject to curvature $R$ (the Ricci tensor) can be calculated by the formula $G_µv = $$8πG over C^4$$ T_µv$



To create a "repulsion" effect, the curvature can be made convex instead of concave. To limit how large an area is affected, use a sharper curve (this will cause the force experience to increase / decrease more rapidly as you approach the peak, instead of the normal $r^-2$ relationship experienced at distance $r$ under standard conditions)



By curving space-time directly, you can also apply the force in any direction instead of just "towards" and "away"






share|improve this answer









$endgroup$








  • 1




    $begingroup$
    As for how that magic works, you change the curvature by changing the population of virtual particles in the intervening space: Changing their density changes c, and changing their dielectric changes G.
    $endgroup$
    – amI
    Mar 16 at 5:26






  • 4




    $begingroup$
    @amI Do you mean $C$? Changing $c$ would be concerning.
    $endgroup$
    – wizzwizz4
    Mar 16 at 11:52










  • $begingroup$
    @wizzwizz4 - Changing C is what massive objects do to the vacuum. Although c is a defined constant, its definition is local. I meant to imply that the ability to manipulate vacuum density without large masses would make c much less universal.
    $endgroup$
    – amI
    Mar 17 at 17:16


















8












$begingroup$


What I'm interested in is not how magic works in this context (assume there is some amount of recharging MP within every person that is used to cast spells), but rather how the gravitational effects caused by the magic could be explained.




If you stitch to the Newtonian model for gravity, you can explain your magic via the ability of controlling magnitude and sign of the masses involved in the gravitational interaction.



The gravitational force between two bodies of mass m at distance r can be calculated according to the famous formula $F=G cdot$$m_1 cdot m_2 over r^2$.



If you can change the sign of m, you can have gravity be repulsive. So, in case you jump from a skyscraper, turning your mass to negative once in fall would have the effect of Earth pushing you up, slowing down your fall. You can also levitate by alternating positive and negative mass values.



Turning upside down a cup while not dropping its content could be achieved in the same way.






share|improve this answer









$endgroup$








  • 1




    $begingroup$
    One thing to consider is that gravity is ridiculously weak compared to any other force. So if you limit the amount you can change the magnitude and sign, you will likely only be able to either crush things or levitate them, if the effect is uniform, since you are always pushing against the earth. Nonuniform gravity can induce some spin in objects, but it will be really hard to generate any sideways motion
    $endgroup$
    – Whitecold
    Mar 15 at 7:38











  • $begingroup$
    Turning your mass to negative would have the effect of you pushing Earth down. Earth would also push you up, but if your inertial mass was also negative that would result in your moving down. (If you're just changing your gravitational mass without changing your inertial mass, you're very strange.)
    $endgroup$
    – wizzwizz4
    Mar 16 at 11:53










  • $begingroup$
    @L.Dutch Is G the acceleration of the earths gravity, so 9.81 m/s^2?
    $endgroup$
    – Tobias F.
    Mar 17 at 13:09







  • 1




    $begingroup$
    @TobiasF. No, it's the gravitational constant (approx 6.7 m^3/[kg * s^2] ) . You're thinking of 'g', which is a recalculation of (G*m/(r^2) with r and m approximately the radius and mass of earth for an item on the surface.
    $endgroup$
    – DonFusili
    Mar 19 at 10:09


















7












$begingroup$

Personally I'd steer clear of trying to get some physics-based in-universe explanation unless your magical society also has a very high technology.



That said...



Since gravity is the result of curvature of space, the simple answer would be to have the magic operate to alter that curvature. We don't know exactly what mechanism causes space to curve in the presence of mass, so you can get away with some artistic licence. It's magic after all.



Let's say that the gravitational field strength, which is an expression of the warping of space, is something that you can move around. You can't make much gravity on your own, but you can potentially take energy from other parts of the field and concentrate it. Reducing the gravity in an area requires you to spread that field energy around, so gravity gets stronger elsewhere. The better you are at controlling it the more complex you can make the resulting changes, and the stronger your magical talent the larger the scope of the changes.



But gravity only really does one thing: pull on stuff. You can put a gravity well in the air 2 feet above the table and watch it fall in, but controlling the orientation of the table is going to be difficult. You'd probably need to create a shaped gravity field the shape of the table to get it in the right orientation, and it's not just going to sit there when you push on it... unless you have enough field strength to counter the push.



Since this is magic though, let's assume that you can deform space the way you want if you have the right spell. You could do all sorts of nasty things with it, like concentrating all of the gravity in a battlefield into a tiny ball of hyper-gravity strong enough to tear humans apart. You could make things fall in any direction, essentially making you the ultimate siege weapon. Water flowing uphill is way simple. How about castles falling into the sky? If it were my castle I'd be doing my best to outlaw gravity magic, as aggressively as possible.



So yes, you can stop people falling. Yes, you can walk on walls and ceilings. But it's going to take a hell of a lot of field shaping to get cups to dance and flip without spilling their contents.



Perhaps what you need is less gravity and more telekinesis? Directly applying forces to objects has a lot of potential too. Maybe a combination of both?






share|improve this answer









$endgroup$




















    5












    $begingroup$

    Gravity sources are everywhere. Everything in the galaxy is attracting everything else.

    The only reason the satellites in space don't come crashing down is because they are traveling fast enough to keep missing the Earth.

    The only reason the Earth doesn't crash into the sun is that it's moving fast enough to keep missing it.

    The only reason the sun doesn't fall into the black hole at the galactic core is that it's moving fast enough to keep missing it.



    If any of them were to lose forward momentum they'd plunge to a fiery death.



    The only reason two apples don't orbit each other is because they are overwhelmed by earths gravity, and the suns gravity, and all of the other gravity sources that influence us.



    So the way that magic controls gravity is by selectively ignoring, reversing, or reinforcing it around whatever thing you want to manipulate. If you need to levitate, reverse the pull of Earths gravity. If you need to push something, reinforce the pull from things in line with the direction you want it to go, and then negate or reverse the pull from the other directions.



    At the highest levels it would be crazy powerful, being able to increase an items gravity field to the point where it becomes neutronium or a singularity.






    share|improve this answer









    $endgroup$




















      3












      $begingroup$

      This is more of an addition to L.Dutch♦'s answer, controlling magnitude and sign of the masses involved in the gravitational interaction, but to achieve some of the points you mentioned (pushing a door out of the way, stopping movement even when affected by an outside force) you need to have a solution for sideways motion and rotation.



      For sideways motion you can take into account the fact that Earth isn't a point source for gravity, instead we are pulled towards each particle that makes up the Earth and our gravity vector is the sum of all those forces. If your character can control the effect of gravity from different sources separately, then sideways motion can be achieved by getting a push from one half of the Earth and getting pulled by the other. You could even use gravity from other celestial bodies (Sun, Jupiter, supermassive black hole at the center of the the Milky Way).



      Rotation can be achieved as described in Whitecold's comment, with nonuniform gravity: have half of the thing you're rotating be pulled down and the other half pulled up (gently so you don't tear it apart). Your rotating cup of water that doesn't spill could be done by having gravity push "into" the cup just near the mouth, pushing from the bottom of the cup to counteract the movement the first push would impart, all the while keeping the cup and water in the air (this might require more skill from your character than simple levitation).






      share|improve this answer









      $endgroup$




















        3












        $begingroup$

        The Higgs field gives elementary particles their mass, and having some sort of a mechanism to attenuate or disable its interactions could work to your benefit at a level that doesn't really have a reasonably obvious counterargument. So in effect, the magic could somehow stop this interaction, making object momentarily more or less massive. (The remote pushing effect you mentioned I found the most difficult to explain...) The intricacies work very much differently for charged leptons and force carrier bosons, though.



        Most of the mass of matter is from the mass of the atomic nuclei, and ≈99% of that mass doesn't come from the mass of the individual elementary particles but the strong force; potential energies of the quarks (and hence protons and neutrons) bound together can be translated to mass via E=mc^2. Another explanation for your magic could thus be not exactly one that works by meddling with gravity but meddling with another fundamental force which affects mass which in turn gets picked up unmodified gravity. The strong force is mediated by gluons (analogous to photons for the electromagnetic interactions), so off we go with gluemancy.



        The problem with this all is of course, that if one is capable enough to modify these tine, fundamental properties of particles, it doesn't easily make much sense to be able to only demonstrate gravity-based powers.






        share|improve this answer









        $endgroup$




















          3












          $begingroup$

          The kind of magic you are talking about is anthropomorphic.




          • Slowing down yourself down when falling, so you don't hit the ground at full speed (as near to 0m/s as possible, at the very least you shouldn't be injured by landing) (e.g. jumping from higher up)

          • Levitating yourself / other objects, so they don't move in any direction

          • Bonus points: still won't move if influenced by another force (e.g. a thrown stone)

          • Pushing objects out of your way (e.g. a locked door)

          • Ability to combine multiple "gravity fields", so you could simultaneously keep a cup in place and rotate it upwards-down, while still keeping the liquid inside



          Gravity is the measurement of the degree to which mass-energy curves space time around it. What you describe above is a bunch of stuff that seems convincing to a human if you described gravity magic to.



          A physics based process won't neatly fit in a "stuff that X magic would do" that a human would narrate, because physics doesn't really care what a human finds narratively convincing.



          As an example, Electro-Magnitism covers everything from being able to touch things, see things, shoot lightning, and pretty much all of chemistry. Meanwhile, the human "story" based Magnatism is about being able to move metals around (because they are "magnetic"), and Electricty magic is about lightning bolts and static electricty.



          If you want anthropomorphic magic, magic that makes sense mythologically or in categories humans care about, it has to be powered by an intelligence with a similar mind to ours. That intelligence somehow set up the rules in ways that we find are pleasing.



          Spells become "instructions" to this intelligence, who then does what asked. The being being interacted with could be a god-like entity, a demon, or a post-singularity AI, or an ancient forgotten society of super-scientists who left "macros" active that their children used to play with.



          In the last case, the affinity of certain people for certain kinds of magic is akin to being able to unlock a certain smart phone with your voice, because it coincidentally is similar to the real owner (it doesn't have to be voice-locked, but you can imagine it being so). The fact that these ancient intelligences let their children play with fireballs would be explained by the fact that everyone in their society was backed up, and repair/resurrection was trivial; that technology doesn't recognize humans as being in their pattern-banks. Humans who can heal are actually using the "veternarian" macros.



          Now, once we divorce the physical mechanism from the "kind of magic", the fact that these are "gravity" macros is because they are the kind of thing that the parent programmed for their kid (or maybe bought the "play with gravity" kit for their kid's system).



          Your entire environment could be a simulation running in an ancient computer, with the original hosts long exinct. The matter you are walking on and breathing could be programmable computronium (mass-energy optimized to do computation) that emulates being matter so as to leave a "natural" experience for growing up, but being computronium this is just an option; "magic" literally ignores the apparent laws of physics, because the laws of physics as you experience them are just an emulation layer.



          Being able to float in either of these cases is little more than a matter of changing the position value of a variable in a specific coordinate space. Now, no human has access to this underlying system; so they cannot insert the wrong arguments and cause the water on the earth to start to orbit the other way around the sun.






          share|improve this answer









          $endgroup$












          • $begingroup$
            "There is no spoon"
            $endgroup$
            – EveryBitHelps
            Mar 16 at 7:26


















          3












          $begingroup$

          I'm borrowing heavily from the "Irregular at the Magic Highschool" light novel series where magic is done by influencing the information bodies of particles. Basically assume everything has an "information body" similar to how in physics simulations every particle has some sort of matrix/vector which stores its properties. These properties can be position, time, velocity, mass, energy, etc... This is somewhat similar to string theory where particles are strings and properties of a particle are determined by the vibration of the string.



          Whichever way you want to go about it, gravity magic influences the vibration/information of the particle related to gravity. There are no "gravity fields" in this solution so I can't check off the ability to create multiple gravity fields but you can still rotate a cup and not have any liquid fall out by influencing the particles of the cup along with the liquid inside.






          share|improve this answer









          $endgroup$




















            0












            $begingroup$


            This answer will not be scientifically detailed. I will be using simple terms to keep it understandable. Assume some accuracy to be lost as a result.




            "What physical law can magic manipulate in order to affect gravity?"



            Gravity Magic Can Manipulate Gravity.



            What? Were you expecting something different?



            Gravity magic works by localizing a mass of mana in a specific region to cause specific gravitational events in that region.



            What effects does Gravity Magic need to have?



            • Gravity Amplification

            • Gravity Reduction

            • Gravity Nullification

            • Gravity Reversion

            So, how would this work?



            First you need to consider what components influence gravity: Mass and Volume. The greater the mass in a volume of space, the greater the gravity it will influence other objects of mass with.



            Mass is the amount of stuff an object is made of. Volume is the amount of space that stuff fills.



            Next, you need to consider how LONG the increase will occur.



            By increasing the mass of an area, you increase its density. In turn, the amount of gravity will increase. Now, normally, gravitational shifts are minor. If you have a crumpled ball of aluminum foil, that's going to have less gravity than a compressed ball of pure aluminum. By very little, but the gravitational amount will be different. But what about if the amount of mass in that ball was increased by 100 despite maintaining the same volume? Then the gravity would increase a fair amount. Would it be noticeable? Not really because the gravity of the planet would be more influential.



            So, how would we affect the changes you desire?



            • Slowed Descent - When you cast Slowed Descent, the caster casts a moving mass of high-density gravity above them which has its own gravity of 0.9 Gs, bringing the influence of the planet's gravity on you to a 10th of what it was.

            • Levitation - When you cast Levitation, the caster casts a mass in an area which attracts anything nearby to its source. The central source of gravity is surrounded by another layer of gravity which serves to keep objects balanced between the two gravitational sources. In order to quickly drag an object, the gravity would be 5Gs, but would decay over time, quickly reaching 1.1 Gs before it maintains that for a while and decays completely. The higher the gravity and more localized the field, the more difficult it would be for an outside force to move the affected objects. The problem is, the greater the gravity, the more dangerous it is to be inside its direct field of influence.

            • Push - A mass of gravity is created which draws in objects towards it.

            • Combined Fields - Just create two separate masses moving at appropriate speeds in different directions so that they are both affecting the same object and not being negatively influenced each other's gravity.

            The answer is just create masses of mana. They'll emit a gravitational force and that force will grant the desired effects. You could even play with the idea of negative mass, but that would be your own choice of creative liberty.






            share|improve this answer











            $endgroup$













              Your Answer








              StackExchange.ready(function()
              var channelOptions =
              tags: "".split(" "),
              id: "579"
              ;
              initTagRenderer("".split(" "), "".split(" "), channelOptions);

              StackExchange.using("externalEditor", function()
              // Have to fire editor after snippets, if snippets enabled
              if (StackExchange.settings.snippets.snippetsEnabled)
              StackExchange.using("snippets", function()
              createEditor();
              );

              else
              createEditor();

              );

              function createEditor()
              StackExchange.prepareEditor(
              heartbeatType: 'answer',
              autoActivateHeartbeat: false,
              convertImagesToLinks: false,
              noModals: true,
              showLowRepImageUploadWarning: true,
              reputationToPostImages: null,
              bindNavPrevention: true,
              postfix: "",
              imageUploader:
              brandingHtml: "Powered by u003ca class="icon-imgur-white" href="https://imgur.com/"u003eu003c/au003e",
              contentPolicyHtml: "User contributions licensed under u003ca href="https://creativecommons.org/licenses/by-sa/3.0/"u003ecc by-sa 3.0 with attribution requiredu003c/au003e u003ca href="https://stackoverflow.com/legal/content-policy"u003e(content policy)u003c/au003e",
              allowUrls: true
              ,
              noCode: true, onDemand: true,
              discardSelector: ".discard-answer"
              ,immediatelyShowMarkdownHelp:true
              );



              );













              draft saved

              draft discarded


















              StackExchange.ready(
              function ()
              StackExchange.openid.initPostLogin('.new-post-login', 'https%3a%2f%2fworldbuilding.stackexchange.com%2fquestions%2f141533%2fgravity-magic-how-does-it-work%23new-answer', 'question_page');

              );

              Post as a guest















              Required, but never shown

























              9 Answers
              9






              active

              oldest

              votes








              9 Answers
              9






              active

              oldest

              votes









              active

              oldest

              votes






              active

              oldest

              votes









              14












              $begingroup$


              What I'm interested in is not how magic works in this context (assume there is some amount of recharging MP within every person that is used to cast spells), but rather how the gravitational effects caused by the magic could be explained.




              If you stick to Einstein's model for gravity, you can explain your magic via the ability of manipulating the curvature of space-time.



              The Gravitational force experienced by an object (the Einstein tensor) subject to curvature $R$ (the Ricci tensor) can be calculated by the formula $G_µv = $$8πG over C^4$$ T_µv$



              To create a "repulsion" effect, the curvature can be made convex instead of concave. To limit how large an area is affected, use a sharper curve (this will cause the force experience to increase / decrease more rapidly as you approach the peak, instead of the normal $r^-2$ relationship experienced at distance $r$ under standard conditions)



              By curving space-time directly, you can also apply the force in any direction instead of just "towards" and "away"






              share|improve this answer









              $endgroup$








              • 1




                $begingroup$
                As for how that magic works, you change the curvature by changing the population of virtual particles in the intervening space: Changing their density changes c, and changing their dielectric changes G.
                $endgroup$
                – amI
                Mar 16 at 5:26






              • 4




                $begingroup$
                @amI Do you mean $C$? Changing $c$ would be concerning.
                $endgroup$
                – wizzwizz4
                Mar 16 at 11:52










              • $begingroup$
                @wizzwizz4 - Changing C is what massive objects do to the vacuum. Although c is a defined constant, its definition is local. I meant to imply that the ability to manipulate vacuum density without large masses would make c much less universal.
                $endgroup$
                – amI
                Mar 17 at 17:16















              14












              $begingroup$


              What I'm interested in is not how magic works in this context (assume there is some amount of recharging MP within every person that is used to cast spells), but rather how the gravitational effects caused by the magic could be explained.




              If you stick to Einstein's model for gravity, you can explain your magic via the ability of manipulating the curvature of space-time.



              The Gravitational force experienced by an object (the Einstein tensor) subject to curvature $R$ (the Ricci tensor) can be calculated by the formula $G_µv = $$8πG over C^4$$ T_µv$



              To create a "repulsion" effect, the curvature can be made convex instead of concave. To limit how large an area is affected, use a sharper curve (this will cause the force experience to increase / decrease more rapidly as you approach the peak, instead of the normal $r^-2$ relationship experienced at distance $r$ under standard conditions)



              By curving space-time directly, you can also apply the force in any direction instead of just "towards" and "away"






              share|improve this answer









              $endgroup$








              • 1




                $begingroup$
                As for how that magic works, you change the curvature by changing the population of virtual particles in the intervening space: Changing their density changes c, and changing their dielectric changes G.
                $endgroup$
                – amI
                Mar 16 at 5:26






              • 4




                $begingroup$
                @amI Do you mean $C$? Changing $c$ would be concerning.
                $endgroup$
                – wizzwizz4
                Mar 16 at 11:52










              • $begingroup$
                @wizzwizz4 - Changing C is what massive objects do to the vacuum. Although c is a defined constant, its definition is local. I meant to imply that the ability to manipulate vacuum density without large masses would make c much less universal.
                $endgroup$
                – amI
                Mar 17 at 17:16













              14












              14








              14





              $begingroup$


              What I'm interested in is not how magic works in this context (assume there is some amount of recharging MP within every person that is used to cast spells), but rather how the gravitational effects caused by the magic could be explained.




              If you stick to Einstein's model for gravity, you can explain your magic via the ability of manipulating the curvature of space-time.



              The Gravitational force experienced by an object (the Einstein tensor) subject to curvature $R$ (the Ricci tensor) can be calculated by the formula $G_µv = $$8πG over C^4$$ T_µv$



              To create a "repulsion" effect, the curvature can be made convex instead of concave. To limit how large an area is affected, use a sharper curve (this will cause the force experience to increase / decrease more rapidly as you approach the peak, instead of the normal $r^-2$ relationship experienced at distance $r$ under standard conditions)



              By curving space-time directly, you can also apply the force in any direction instead of just "towards" and "away"






              share|improve this answer









              $endgroup$




              What I'm interested in is not how magic works in this context (assume there is some amount of recharging MP within every person that is used to cast spells), but rather how the gravitational effects caused by the magic could be explained.




              If you stick to Einstein's model for gravity, you can explain your magic via the ability of manipulating the curvature of space-time.



              The Gravitational force experienced by an object (the Einstein tensor) subject to curvature $R$ (the Ricci tensor) can be calculated by the formula $G_µv = $$8πG over C^4$$ T_µv$



              To create a "repulsion" effect, the curvature can be made convex instead of concave. To limit how large an area is affected, use a sharper curve (this will cause the force experience to increase / decrease more rapidly as you approach the peak, instead of the normal $r^-2$ relationship experienced at distance $r$ under standard conditions)



              By curving space-time directly, you can also apply the force in any direction instead of just "towards" and "away"







              share|improve this answer












              share|improve this answer



              share|improve this answer










              answered Mar 15 at 10:44









              ChronocidalChronocidal

              7,13711035




              7,13711035







              • 1




                $begingroup$
                As for how that magic works, you change the curvature by changing the population of virtual particles in the intervening space: Changing their density changes c, and changing their dielectric changes G.
                $endgroup$
                – amI
                Mar 16 at 5:26






              • 4




                $begingroup$
                @amI Do you mean $C$? Changing $c$ would be concerning.
                $endgroup$
                – wizzwizz4
                Mar 16 at 11:52










              • $begingroup$
                @wizzwizz4 - Changing C is what massive objects do to the vacuum. Although c is a defined constant, its definition is local. I meant to imply that the ability to manipulate vacuum density without large masses would make c much less universal.
                $endgroup$
                – amI
                Mar 17 at 17:16












              • 1




                $begingroup$
                As for how that magic works, you change the curvature by changing the population of virtual particles in the intervening space: Changing their density changes c, and changing their dielectric changes G.
                $endgroup$
                – amI
                Mar 16 at 5:26






              • 4




                $begingroup$
                @amI Do you mean $C$? Changing $c$ would be concerning.
                $endgroup$
                – wizzwizz4
                Mar 16 at 11:52










              • $begingroup$
                @wizzwizz4 - Changing C is what massive objects do to the vacuum. Although c is a defined constant, its definition is local. I meant to imply that the ability to manipulate vacuum density without large masses would make c much less universal.
                $endgroup$
                – amI
                Mar 17 at 17:16







              1




              1




              $begingroup$
              As for how that magic works, you change the curvature by changing the population of virtual particles in the intervening space: Changing their density changes c, and changing their dielectric changes G.
              $endgroup$
              – amI
              Mar 16 at 5:26




              $begingroup$
              As for how that magic works, you change the curvature by changing the population of virtual particles in the intervening space: Changing their density changes c, and changing their dielectric changes G.
              $endgroup$
              – amI
              Mar 16 at 5:26




              4




              4




              $begingroup$
              @amI Do you mean $C$? Changing $c$ would be concerning.
              $endgroup$
              – wizzwizz4
              Mar 16 at 11:52




              $begingroup$
              @amI Do you mean $C$? Changing $c$ would be concerning.
              $endgroup$
              – wizzwizz4
              Mar 16 at 11:52












              $begingroup$
              @wizzwizz4 - Changing C is what massive objects do to the vacuum. Although c is a defined constant, its definition is local. I meant to imply that the ability to manipulate vacuum density without large masses would make c much less universal.
              $endgroup$
              – amI
              Mar 17 at 17:16




              $begingroup$
              @wizzwizz4 - Changing C is what massive objects do to the vacuum. Although c is a defined constant, its definition is local. I meant to imply that the ability to manipulate vacuum density without large masses would make c much less universal.
              $endgroup$
              – amI
              Mar 17 at 17:16











              8












              $begingroup$


              What I'm interested in is not how magic works in this context (assume there is some amount of recharging MP within every person that is used to cast spells), but rather how the gravitational effects caused by the magic could be explained.




              If you stitch to the Newtonian model for gravity, you can explain your magic via the ability of controlling magnitude and sign of the masses involved in the gravitational interaction.



              The gravitational force between two bodies of mass m at distance r can be calculated according to the famous formula $F=G cdot$$m_1 cdot m_2 over r^2$.



              If you can change the sign of m, you can have gravity be repulsive. So, in case you jump from a skyscraper, turning your mass to negative once in fall would have the effect of Earth pushing you up, slowing down your fall. You can also levitate by alternating positive and negative mass values.



              Turning upside down a cup while not dropping its content could be achieved in the same way.






              share|improve this answer









              $endgroup$








              • 1




                $begingroup$
                One thing to consider is that gravity is ridiculously weak compared to any other force. So if you limit the amount you can change the magnitude and sign, you will likely only be able to either crush things or levitate them, if the effect is uniform, since you are always pushing against the earth. Nonuniform gravity can induce some spin in objects, but it will be really hard to generate any sideways motion
                $endgroup$
                – Whitecold
                Mar 15 at 7:38











              • $begingroup$
                Turning your mass to negative would have the effect of you pushing Earth down. Earth would also push you up, but if your inertial mass was also negative that would result in your moving down. (If you're just changing your gravitational mass without changing your inertial mass, you're very strange.)
                $endgroup$
                – wizzwizz4
                Mar 16 at 11:53










              • $begingroup$
                @L.Dutch Is G the acceleration of the earths gravity, so 9.81 m/s^2?
                $endgroup$
                – Tobias F.
                Mar 17 at 13:09







              • 1




                $begingroup$
                @TobiasF. No, it's the gravitational constant (approx 6.7 m^3/[kg * s^2] ) . You're thinking of 'g', which is a recalculation of (G*m/(r^2) with r and m approximately the radius and mass of earth for an item on the surface.
                $endgroup$
                – DonFusili
                Mar 19 at 10:09















              8












              $begingroup$


              What I'm interested in is not how magic works in this context (assume there is some amount of recharging MP within every person that is used to cast spells), but rather how the gravitational effects caused by the magic could be explained.




              If you stitch to the Newtonian model for gravity, you can explain your magic via the ability of controlling magnitude and sign of the masses involved in the gravitational interaction.



              The gravitational force between two bodies of mass m at distance r can be calculated according to the famous formula $F=G cdot$$m_1 cdot m_2 over r^2$.



              If you can change the sign of m, you can have gravity be repulsive. So, in case you jump from a skyscraper, turning your mass to negative once in fall would have the effect of Earth pushing you up, slowing down your fall. You can also levitate by alternating positive and negative mass values.



              Turning upside down a cup while not dropping its content could be achieved in the same way.






              share|improve this answer









              $endgroup$








              • 1




                $begingroup$
                One thing to consider is that gravity is ridiculously weak compared to any other force. So if you limit the amount you can change the magnitude and sign, you will likely only be able to either crush things or levitate them, if the effect is uniform, since you are always pushing against the earth. Nonuniform gravity can induce some spin in objects, but it will be really hard to generate any sideways motion
                $endgroup$
                – Whitecold
                Mar 15 at 7:38











              • $begingroup$
                Turning your mass to negative would have the effect of you pushing Earth down. Earth would also push you up, but if your inertial mass was also negative that would result in your moving down. (If you're just changing your gravitational mass without changing your inertial mass, you're very strange.)
                $endgroup$
                – wizzwizz4
                Mar 16 at 11:53










              • $begingroup$
                @L.Dutch Is G the acceleration of the earths gravity, so 9.81 m/s^2?
                $endgroup$
                – Tobias F.
                Mar 17 at 13:09







              • 1




                $begingroup$
                @TobiasF. No, it's the gravitational constant (approx 6.7 m^3/[kg * s^2] ) . You're thinking of 'g', which is a recalculation of (G*m/(r^2) with r and m approximately the radius and mass of earth for an item on the surface.
                $endgroup$
                – DonFusili
                Mar 19 at 10:09













              8












              8








              8





              $begingroup$


              What I'm interested in is not how magic works in this context (assume there is some amount of recharging MP within every person that is used to cast spells), but rather how the gravitational effects caused by the magic could be explained.




              If you stitch to the Newtonian model for gravity, you can explain your magic via the ability of controlling magnitude and sign of the masses involved in the gravitational interaction.



              The gravitational force between two bodies of mass m at distance r can be calculated according to the famous formula $F=G cdot$$m_1 cdot m_2 over r^2$.



              If you can change the sign of m, you can have gravity be repulsive. So, in case you jump from a skyscraper, turning your mass to negative once in fall would have the effect of Earth pushing you up, slowing down your fall. You can also levitate by alternating positive and negative mass values.



              Turning upside down a cup while not dropping its content could be achieved in the same way.






              share|improve this answer









              $endgroup$




              What I'm interested in is not how magic works in this context (assume there is some amount of recharging MP within every person that is used to cast spells), but rather how the gravitational effects caused by the magic could be explained.




              If you stitch to the Newtonian model for gravity, you can explain your magic via the ability of controlling magnitude and sign of the masses involved in the gravitational interaction.



              The gravitational force between two bodies of mass m at distance r can be calculated according to the famous formula $F=G cdot$$m_1 cdot m_2 over r^2$.



              If you can change the sign of m, you can have gravity be repulsive. So, in case you jump from a skyscraper, turning your mass to negative once in fall would have the effect of Earth pushing you up, slowing down your fall. You can also levitate by alternating positive and negative mass values.



              Turning upside down a cup while not dropping its content could be achieved in the same way.







              share|improve this answer












              share|improve this answer



              share|improve this answer










              answered Mar 15 at 7:14









              L.DutchL.Dutch

              91.3k29211439




              91.3k29211439







              • 1




                $begingroup$
                One thing to consider is that gravity is ridiculously weak compared to any other force. So if you limit the amount you can change the magnitude and sign, you will likely only be able to either crush things or levitate them, if the effect is uniform, since you are always pushing against the earth. Nonuniform gravity can induce some spin in objects, but it will be really hard to generate any sideways motion
                $endgroup$
                – Whitecold
                Mar 15 at 7:38











              • $begingroup$
                Turning your mass to negative would have the effect of you pushing Earth down. Earth would also push you up, but if your inertial mass was also negative that would result in your moving down. (If you're just changing your gravitational mass without changing your inertial mass, you're very strange.)
                $endgroup$
                – wizzwizz4
                Mar 16 at 11:53










              • $begingroup$
                @L.Dutch Is G the acceleration of the earths gravity, so 9.81 m/s^2?
                $endgroup$
                – Tobias F.
                Mar 17 at 13:09







              • 1




                $begingroup$
                @TobiasF. No, it's the gravitational constant (approx 6.7 m^3/[kg * s^2] ) . You're thinking of 'g', which is a recalculation of (G*m/(r^2) with r and m approximately the radius and mass of earth for an item on the surface.
                $endgroup$
                – DonFusili
                Mar 19 at 10:09












              • 1




                $begingroup$
                One thing to consider is that gravity is ridiculously weak compared to any other force. So if you limit the amount you can change the magnitude and sign, you will likely only be able to either crush things or levitate them, if the effect is uniform, since you are always pushing against the earth. Nonuniform gravity can induce some spin in objects, but it will be really hard to generate any sideways motion
                $endgroup$
                – Whitecold
                Mar 15 at 7:38











              • $begingroup$
                Turning your mass to negative would have the effect of you pushing Earth down. Earth would also push you up, but if your inertial mass was also negative that would result in your moving down. (If you're just changing your gravitational mass without changing your inertial mass, you're very strange.)
                $endgroup$
                – wizzwizz4
                Mar 16 at 11:53










              • $begingroup$
                @L.Dutch Is G the acceleration of the earths gravity, so 9.81 m/s^2?
                $endgroup$
                – Tobias F.
                Mar 17 at 13:09







              • 1




                $begingroup$
                @TobiasF. No, it's the gravitational constant (approx 6.7 m^3/[kg * s^2] ) . You're thinking of 'g', which is a recalculation of (G*m/(r^2) with r and m approximately the radius and mass of earth for an item on the surface.
                $endgroup$
                – DonFusili
                Mar 19 at 10:09







              1




              1




              $begingroup$
              One thing to consider is that gravity is ridiculously weak compared to any other force. So if you limit the amount you can change the magnitude and sign, you will likely only be able to either crush things or levitate them, if the effect is uniform, since you are always pushing against the earth. Nonuniform gravity can induce some spin in objects, but it will be really hard to generate any sideways motion
              $endgroup$
              – Whitecold
              Mar 15 at 7:38





              $begingroup$
              One thing to consider is that gravity is ridiculously weak compared to any other force. So if you limit the amount you can change the magnitude and sign, you will likely only be able to either crush things or levitate them, if the effect is uniform, since you are always pushing against the earth. Nonuniform gravity can induce some spin in objects, but it will be really hard to generate any sideways motion
              $endgroup$
              – Whitecold
              Mar 15 at 7:38













              $begingroup$
              Turning your mass to negative would have the effect of you pushing Earth down. Earth would also push you up, but if your inertial mass was also negative that would result in your moving down. (If you're just changing your gravitational mass without changing your inertial mass, you're very strange.)
              $endgroup$
              – wizzwizz4
              Mar 16 at 11:53




              $begingroup$
              Turning your mass to negative would have the effect of you pushing Earth down. Earth would also push you up, but if your inertial mass was also negative that would result in your moving down. (If you're just changing your gravitational mass without changing your inertial mass, you're very strange.)
              $endgroup$
              – wizzwizz4
              Mar 16 at 11:53












              $begingroup$
              @L.Dutch Is G the acceleration of the earths gravity, so 9.81 m/s^2?
              $endgroup$
              – Tobias F.
              Mar 17 at 13:09





              $begingroup$
              @L.Dutch Is G the acceleration of the earths gravity, so 9.81 m/s^2?
              $endgroup$
              – Tobias F.
              Mar 17 at 13:09





              1




              1




              $begingroup$
              @TobiasF. No, it's the gravitational constant (approx 6.7 m^3/[kg * s^2] ) . You're thinking of 'g', which is a recalculation of (G*m/(r^2) with r and m approximately the radius and mass of earth for an item on the surface.
              $endgroup$
              – DonFusili
              Mar 19 at 10:09




              $begingroup$
              @TobiasF. No, it's the gravitational constant (approx 6.7 m^3/[kg * s^2] ) . You're thinking of 'g', which is a recalculation of (G*m/(r^2) with r and m approximately the radius and mass of earth for an item on the surface.
              $endgroup$
              – DonFusili
              Mar 19 at 10:09











              7












              $begingroup$

              Personally I'd steer clear of trying to get some physics-based in-universe explanation unless your magical society also has a very high technology.



              That said...



              Since gravity is the result of curvature of space, the simple answer would be to have the magic operate to alter that curvature. We don't know exactly what mechanism causes space to curve in the presence of mass, so you can get away with some artistic licence. It's magic after all.



              Let's say that the gravitational field strength, which is an expression of the warping of space, is something that you can move around. You can't make much gravity on your own, but you can potentially take energy from other parts of the field and concentrate it. Reducing the gravity in an area requires you to spread that field energy around, so gravity gets stronger elsewhere. The better you are at controlling it the more complex you can make the resulting changes, and the stronger your magical talent the larger the scope of the changes.



              But gravity only really does one thing: pull on stuff. You can put a gravity well in the air 2 feet above the table and watch it fall in, but controlling the orientation of the table is going to be difficult. You'd probably need to create a shaped gravity field the shape of the table to get it in the right orientation, and it's not just going to sit there when you push on it... unless you have enough field strength to counter the push.



              Since this is magic though, let's assume that you can deform space the way you want if you have the right spell. You could do all sorts of nasty things with it, like concentrating all of the gravity in a battlefield into a tiny ball of hyper-gravity strong enough to tear humans apart. You could make things fall in any direction, essentially making you the ultimate siege weapon. Water flowing uphill is way simple. How about castles falling into the sky? If it were my castle I'd be doing my best to outlaw gravity magic, as aggressively as possible.



              So yes, you can stop people falling. Yes, you can walk on walls and ceilings. But it's going to take a hell of a lot of field shaping to get cups to dance and flip without spilling their contents.



              Perhaps what you need is less gravity and more telekinesis? Directly applying forces to objects has a lot of potential too. Maybe a combination of both?






              share|improve this answer









              $endgroup$

















                7












                $begingroup$

                Personally I'd steer clear of trying to get some physics-based in-universe explanation unless your magical society also has a very high technology.



                That said...



                Since gravity is the result of curvature of space, the simple answer would be to have the magic operate to alter that curvature. We don't know exactly what mechanism causes space to curve in the presence of mass, so you can get away with some artistic licence. It's magic after all.



                Let's say that the gravitational field strength, which is an expression of the warping of space, is something that you can move around. You can't make much gravity on your own, but you can potentially take energy from other parts of the field and concentrate it. Reducing the gravity in an area requires you to spread that field energy around, so gravity gets stronger elsewhere. The better you are at controlling it the more complex you can make the resulting changes, and the stronger your magical talent the larger the scope of the changes.



                But gravity only really does one thing: pull on stuff. You can put a gravity well in the air 2 feet above the table and watch it fall in, but controlling the orientation of the table is going to be difficult. You'd probably need to create a shaped gravity field the shape of the table to get it in the right orientation, and it's not just going to sit there when you push on it... unless you have enough field strength to counter the push.



                Since this is magic though, let's assume that you can deform space the way you want if you have the right spell. You could do all sorts of nasty things with it, like concentrating all of the gravity in a battlefield into a tiny ball of hyper-gravity strong enough to tear humans apart. You could make things fall in any direction, essentially making you the ultimate siege weapon. Water flowing uphill is way simple. How about castles falling into the sky? If it were my castle I'd be doing my best to outlaw gravity magic, as aggressively as possible.



                So yes, you can stop people falling. Yes, you can walk on walls and ceilings. But it's going to take a hell of a lot of field shaping to get cups to dance and flip without spilling their contents.



                Perhaps what you need is less gravity and more telekinesis? Directly applying forces to objects has a lot of potential too. Maybe a combination of both?






                share|improve this answer









                $endgroup$















                  7












                  7








                  7





                  $begingroup$

                  Personally I'd steer clear of trying to get some physics-based in-universe explanation unless your magical society also has a very high technology.



                  That said...



                  Since gravity is the result of curvature of space, the simple answer would be to have the magic operate to alter that curvature. We don't know exactly what mechanism causes space to curve in the presence of mass, so you can get away with some artistic licence. It's magic after all.



                  Let's say that the gravitational field strength, which is an expression of the warping of space, is something that you can move around. You can't make much gravity on your own, but you can potentially take energy from other parts of the field and concentrate it. Reducing the gravity in an area requires you to spread that field energy around, so gravity gets stronger elsewhere. The better you are at controlling it the more complex you can make the resulting changes, and the stronger your magical talent the larger the scope of the changes.



                  But gravity only really does one thing: pull on stuff. You can put a gravity well in the air 2 feet above the table and watch it fall in, but controlling the orientation of the table is going to be difficult. You'd probably need to create a shaped gravity field the shape of the table to get it in the right orientation, and it's not just going to sit there when you push on it... unless you have enough field strength to counter the push.



                  Since this is magic though, let's assume that you can deform space the way you want if you have the right spell. You could do all sorts of nasty things with it, like concentrating all of the gravity in a battlefield into a tiny ball of hyper-gravity strong enough to tear humans apart. You could make things fall in any direction, essentially making you the ultimate siege weapon. Water flowing uphill is way simple. How about castles falling into the sky? If it were my castle I'd be doing my best to outlaw gravity magic, as aggressively as possible.



                  So yes, you can stop people falling. Yes, you can walk on walls and ceilings. But it's going to take a hell of a lot of field shaping to get cups to dance and flip without spilling their contents.



                  Perhaps what you need is less gravity and more telekinesis? Directly applying forces to objects has a lot of potential too. Maybe a combination of both?






                  share|improve this answer









                  $endgroup$



                  Personally I'd steer clear of trying to get some physics-based in-universe explanation unless your magical society also has a very high technology.



                  That said...



                  Since gravity is the result of curvature of space, the simple answer would be to have the magic operate to alter that curvature. We don't know exactly what mechanism causes space to curve in the presence of mass, so you can get away with some artistic licence. It's magic after all.



                  Let's say that the gravitational field strength, which is an expression of the warping of space, is something that you can move around. You can't make much gravity on your own, but you can potentially take energy from other parts of the field and concentrate it. Reducing the gravity in an area requires you to spread that field energy around, so gravity gets stronger elsewhere. The better you are at controlling it the more complex you can make the resulting changes, and the stronger your magical talent the larger the scope of the changes.



                  But gravity only really does one thing: pull on stuff. You can put a gravity well in the air 2 feet above the table and watch it fall in, but controlling the orientation of the table is going to be difficult. You'd probably need to create a shaped gravity field the shape of the table to get it in the right orientation, and it's not just going to sit there when you push on it... unless you have enough field strength to counter the push.



                  Since this is magic though, let's assume that you can deform space the way you want if you have the right spell. You could do all sorts of nasty things with it, like concentrating all of the gravity in a battlefield into a tiny ball of hyper-gravity strong enough to tear humans apart. You could make things fall in any direction, essentially making you the ultimate siege weapon. Water flowing uphill is way simple. How about castles falling into the sky? If it were my castle I'd be doing my best to outlaw gravity magic, as aggressively as possible.



                  So yes, you can stop people falling. Yes, you can walk on walls and ceilings. But it's going to take a hell of a lot of field shaping to get cups to dance and flip without spilling their contents.



                  Perhaps what you need is less gravity and more telekinesis? Directly applying forces to objects has a lot of potential too. Maybe a combination of both?







                  share|improve this answer












                  share|improve this answer



                  share|improve this answer










                  answered Mar 15 at 12:02









                  CoreyCorey

                  974411




                  974411





















                      5












                      $begingroup$

                      Gravity sources are everywhere. Everything in the galaxy is attracting everything else.

                      The only reason the satellites in space don't come crashing down is because they are traveling fast enough to keep missing the Earth.

                      The only reason the Earth doesn't crash into the sun is that it's moving fast enough to keep missing it.

                      The only reason the sun doesn't fall into the black hole at the galactic core is that it's moving fast enough to keep missing it.



                      If any of them were to lose forward momentum they'd plunge to a fiery death.



                      The only reason two apples don't orbit each other is because they are overwhelmed by earths gravity, and the suns gravity, and all of the other gravity sources that influence us.



                      So the way that magic controls gravity is by selectively ignoring, reversing, or reinforcing it around whatever thing you want to manipulate. If you need to levitate, reverse the pull of Earths gravity. If you need to push something, reinforce the pull from things in line with the direction you want it to go, and then negate or reverse the pull from the other directions.



                      At the highest levels it would be crazy powerful, being able to increase an items gravity field to the point where it becomes neutronium or a singularity.






                      share|improve this answer









                      $endgroup$

















                        5












                        $begingroup$

                        Gravity sources are everywhere. Everything in the galaxy is attracting everything else.

                        The only reason the satellites in space don't come crashing down is because they are traveling fast enough to keep missing the Earth.

                        The only reason the Earth doesn't crash into the sun is that it's moving fast enough to keep missing it.

                        The only reason the sun doesn't fall into the black hole at the galactic core is that it's moving fast enough to keep missing it.



                        If any of them were to lose forward momentum they'd plunge to a fiery death.



                        The only reason two apples don't orbit each other is because they are overwhelmed by earths gravity, and the suns gravity, and all of the other gravity sources that influence us.



                        So the way that magic controls gravity is by selectively ignoring, reversing, or reinforcing it around whatever thing you want to manipulate. If you need to levitate, reverse the pull of Earths gravity. If you need to push something, reinforce the pull from things in line with the direction you want it to go, and then negate or reverse the pull from the other directions.



                        At the highest levels it would be crazy powerful, being able to increase an items gravity field to the point where it becomes neutronium or a singularity.






                        share|improve this answer









                        $endgroup$















                          5












                          5








                          5





                          $begingroup$

                          Gravity sources are everywhere. Everything in the galaxy is attracting everything else.

                          The only reason the satellites in space don't come crashing down is because they are traveling fast enough to keep missing the Earth.

                          The only reason the Earth doesn't crash into the sun is that it's moving fast enough to keep missing it.

                          The only reason the sun doesn't fall into the black hole at the galactic core is that it's moving fast enough to keep missing it.



                          If any of them were to lose forward momentum they'd plunge to a fiery death.



                          The only reason two apples don't orbit each other is because they are overwhelmed by earths gravity, and the suns gravity, and all of the other gravity sources that influence us.



                          So the way that magic controls gravity is by selectively ignoring, reversing, or reinforcing it around whatever thing you want to manipulate. If you need to levitate, reverse the pull of Earths gravity. If you need to push something, reinforce the pull from things in line with the direction you want it to go, and then negate or reverse the pull from the other directions.



                          At the highest levels it would be crazy powerful, being able to increase an items gravity field to the point where it becomes neutronium or a singularity.






                          share|improve this answer









                          $endgroup$



                          Gravity sources are everywhere. Everything in the galaxy is attracting everything else.

                          The only reason the satellites in space don't come crashing down is because they are traveling fast enough to keep missing the Earth.

                          The only reason the Earth doesn't crash into the sun is that it's moving fast enough to keep missing it.

                          The only reason the sun doesn't fall into the black hole at the galactic core is that it's moving fast enough to keep missing it.



                          If any of them were to lose forward momentum they'd plunge to a fiery death.



                          The only reason two apples don't orbit each other is because they are overwhelmed by earths gravity, and the suns gravity, and all of the other gravity sources that influence us.



                          So the way that magic controls gravity is by selectively ignoring, reversing, or reinforcing it around whatever thing you want to manipulate. If you need to levitate, reverse the pull of Earths gravity. If you need to push something, reinforce the pull from things in line with the direction you want it to go, and then negate or reverse the pull from the other directions.



                          At the highest levels it would be crazy powerful, being able to increase an items gravity field to the point where it becomes neutronium or a singularity.







                          share|improve this answer












                          share|improve this answer



                          share|improve this answer










                          answered Mar 15 at 13:00









                          AndyD273AndyD273

                          31.2k258137




                          31.2k258137





















                              3












                              $begingroup$

                              This is more of an addition to L.Dutch♦'s answer, controlling magnitude and sign of the masses involved in the gravitational interaction, but to achieve some of the points you mentioned (pushing a door out of the way, stopping movement even when affected by an outside force) you need to have a solution for sideways motion and rotation.



                              For sideways motion you can take into account the fact that Earth isn't a point source for gravity, instead we are pulled towards each particle that makes up the Earth and our gravity vector is the sum of all those forces. If your character can control the effect of gravity from different sources separately, then sideways motion can be achieved by getting a push from one half of the Earth and getting pulled by the other. You could even use gravity from other celestial bodies (Sun, Jupiter, supermassive black hole at the center of the the Milky Way).



                              Rotation can be achieved as described in Whitecold's comment, with nonuniform gravity: have half of the thing you're rotating be pulled down and the other half pulled up (gently so you don't tear it apart). Your rotating cup of water that doesn't spill could be done by having gravity push "into" the cup just near the mouth, pushing from the bottom of the cup to counteract the movement the first push would impart, all the while keeping the cup and water in the air (this might require more skill from your character than simple levitation).






                              share|improve this answer









                              $endgroup$

















                                3












                                $begingroup$

                                This is more of an addition to L.Dutch♦'s answer, controlling magnitude and sign of the masses involved in the gravitational interaction, but to achieve some of the points you mentioned (pushing a door out of the way, stopping movement even when affected by an outside force) you need to have a solution for sideways motion and rotation.



                                For sideways motion you can take into account the fact that Earth isn't a point source for gravity, instead we are pulled towards each particle that makes up the Earth and our gravity vector is the sum of all those forces. If your character can control the effect of gravity from different sources separately, then sideways motion can be achieved by getting a push from one half of the Earth and getting pulled by the other. You could even use gravity from other celestial bodies (Sun, Jupiter, supermassive black hole at the center of the the Milky Way).



                                Rotation can be achieved as described in Whitecold's comment, with nonuniform gravity: have half of the thing you're rotating be pulled down and the other half pulled up (gently so you don't tear it apart). Your rotating cup of water that doesn't spill could be done by having gravity push "into" the cup just near the mouth, pushing from the bottom of the cup to counteract the movement the first push would impart, all the while keeping the cup and water in the air (this might require more skill from your character than simple levitation).






                                share|improve this answer









                                $endgroup$















                                  3












                                  3








                                  3





                                  $begingroup$

                                  This is more of an addition to L.Dutch♦'s answer, controlling magnitude and sign of the masses involved in the gravitational interaction, but to achieve some of the points you mentioned (pushing a door out of the way, stopping movement even when affected by an outside force) you need to have a solution for sideways motion and rotation.



                                  For sideways motion you can take into account the fact that Earth isn't a point source for gravity, instead we are pulled towards each particle that makes up the Earth and our gravity vector is the sum of all those forces. If your character can control the effect of gravity from different sources separately, then sideways motion can be achieved by getting a push from one half of the Earth and getting pulled by the other. You could even use gravity from other celestial bodies (Sun, Jupiter, supermassive black hole at the center of the the Milky Way).



                                  Rotation can be achieved as described in Whitecold's comment, with nonuniform gravity: have half of the thing you're rotating be pulled down and the other half pulled up (gently so you don't tear it apart). Your rotating cup of water that doesn't spill could be done by having gravity push "into" the cup just near the mouth, pushing from the bottom of the cup to counteract the movement the first push would impart, all the while keeping the cup and water in the air (this might require more skill from your character than simple levitation).






                                  share|improve this answer









                                  $endgroup$



                                  This is more of an addition to L.Dutch♦'s answer, controlling magnitude and sign of the masses involved in the gravitational interaction, but to achieve some of the points you mentioned (pushing a door out of the way, stopping movement even when affected by an outside force) you need to have a solution for sideways motion and rotation.



                                  For sideways motion you can take into account the fact that Earth isn't a point source for gravity, instead we are pulled towards each particle that makes up the Earth and our gravity vector is the sum of all those forces. If your character can control the effect of gravity from different sources separately, then sideways motion can be achieved by getting a push from one half of the Earth and getting pulled by the other. You could even use gravity from other celestial bodies (Sun, Jupiter, supermassive black hole at the center of the the Milky Way).



                                  Rotation can be achieved as described in Whitecold's comment, with nonuniform gravity: have half of the thing you're rotating be pulled down and the other half pulled up (gently so you don't tear it apart). Your rotating cup of water that doesn't spill could be done by having gravity push "into" the cup just near the mouth, pushing from the bottom of the cup to counteract the movement the first push would impart, all the while keeping the cup and water in the air (this might require more skill from your character than simple levitation).







                                  share|improve this answer












                                  share|improve this answer



                                  share|improve this answer










                                  answered Mar 15 at 12:02









                                  JCPJCP

                                  313




                                  313





















                                      3












                                      $begingroup$

                                      The Higgs field gives elementary particles their mass, and having some sort of a mechanism to attenuate or disable its interactions could work to your benefit at a level that doesn't really have a reasonably obvious counterargument. So in effect, the magic could somehow stop this interaction, making object momentarily more or less massive. (The remote pushing effect you mentioned I found the most difficult to explain...) The intricacies work very much differently for charged leptons and force carrier bosons, though.



                                      Most of the mass of matter is from the mass of the atomic nuclei, and ≈99% of that mass doesn't come from the mass of the individual elementary particles but the strong force; potential energies of the quarks (and hence protons and neutrons) bound together can be translated to mass via E=mc^2. Another explanation for your magic could thus be not exactly one that works by meddling with gravity but meddling with another fundamental force which affects mass which in turn gets picked up unmodified gravity. The strong force is mediated by gluons (analogous to photons for the electromagnetic interactions), so off we go with gluemancy.



                                      The problem with this all is of course, that if one is capable enough to modify these tine, fundamental properties of particles, it doesn't easily make much sense to be able to only demonstrate gravity-based powers.






                                      share|improve this answer









                                      $endgroup$

















                                        3












                                        $begingroup$

                                        The Higgs field gives elementary particles their mass, and having some sort of a mechanism to attenuate or disable its interactions could work to your benefit at a level that doesn't really have a reasonably obvious counterargument. So in effect, the magic could somehow stop this interaction, making object momentarily more or less massive. (The remote pushing effect you mentioned I found the most difficult to explain...) The intricacies work very much differently for charged leptons and force carrier bosons, though.



                                        Most of the mass of matter is from the mass of the atomic nuclei, and ≈99% of that mass doesn't come from the mass of the individual elementary particles but the strong force; potential energies of the quarks (and hence protons and neutrons) bound together can be translated to mass via E=mc^2. Another explanation for your magic could thus be not exactly one that works by meddling with gravity but meddling with another fundamental force which affects mass which in turn gets picked up unmodified gravity. The strong force is mediated by gluons (analogous to photons for the electromagnetic interactions), so off we go with gluemancy.



                                        The problem with this all is of course, that if one is capable enough to modify these tine, fundamental properties of particles, it doesn't easily make much sense to be able to only demonstrate gravity-based powers.






                                        share|improve this answer









                                        $endgroup$















                                          3












                                          3








                                          3





                                          $begingroup$

                                          The Higgs field gives elementary particles their mass, and having some sort of a mechanism to attenuate or disable its interactions could work to your benefit at a level that doesn't really have a reasonably obvious counterargument. So in effect, the magic could somehow stop this interaction, making object momentarily more or less massive. (The remote pushing effect you mentioned I found the most difficult to explain...) The intricacies work very much differently for charged leptons and force carrier bosons, though.



                                          Most of the mass of matter is from the mass of the atomic nuclei, and ≈99% of that mass doesn't come from the mass of the individual elementary particles but the strong force; potential energies of the quarks (and hence protons and neutrons) bound together can be translated to mass via E=mc^2. Another explanation for your magic could thus be not exactly one that works by meddling with gravity but meddling with another fundamental force which affects mass which in turn gets picked up unmodified gravity. The strong force is mediated by gluons (analogous to photons for the electromagnetic interactions), so off we go with gluemancy.



                                          The problem with this all is of course, that if one is capable enough to modify these tine, fundamental properties of particles, it doesn't easily make much sense to be able to only demonstrate gravity-based powers.






                                          share|improve this answer









                                          $endgroup$



                                          The Higgs field gives elementary particles their mass, and having some sort of a mechanism to attenuate or disable its interactions could work to your benefit at a level that doesn't really have a reasonably obvious counterargument. So in effect, the magic could somehow stop this interaction, making object momentarily more or less massive. (The remote pushing effect you mentioned I found the most difficult to explain...) The intricacies work very much differently for charged leptons and force carrier bosons, though.



                                          Most of the mass of matter is from the mass of the atomic nuclei, and ≈99% of that mass doesn't come from the mass of the individual elementary particles but the strong force; potential energies of the quarks (and hence protons and neutrons) bound together can be translated to mass via E=mc^2. Another explanation for your magic could thus be not exactly one that works by meddling with gravity but meddling with another fundamental force which affects mass which in turn gets picked up unmodified gravity. The strong force is mediated by gluons (analogous to photons for the electromagnetic interactions), so off we go with gluemancy.



                                          The problem with this all is of course, that if one is capable enough to modify these tine, fundamental properties of particles, it doesn't easily make much sense to be able to only demonstrate gravity-based powers.







                                          share|improve this answer












                                          share|improve this answer



                                          share|improve this answer










                                          answered Mar 15 at 14:07









                                          JoonasD6JoonasD6

                                          311




                                          311





















                                              3












                                              $begingroup$

                                              The kind of magic you are talking about is anthropomorphic.




                                              • Slowing down yourself down when falling, so you don't hit the ground at full speed (as near to 0m/s as possible, at the very least you shouldn't be injured by landing) (e.g. jumping from higher up)

                                              • Levitating yourself / other objects, so they don't move in any direction

                                              • Bonus points: still won't move if influenced by another force (e.g. a thrown stone)

                                              • Pushing objects out of your way (e.g. a locked door)

                                              • Ability to combine multiple "gravity fields", so you could simultaneously keep a cup in place and rotate it upwards-down, while still keeping the liquid inside



                                              Gravity is the measurement of the degree to which mass-energy curves space time around it. What you describe above is a bunch of stuff that seems convincing to a human if you described gravity magic to.



                                              A physics based process won't neatly fit in a "stuff that X magic would do" that a human would narrate, because physics doesn't really care what a human finds narratively convincing.



                                              As an example, Electro-Magnitism covers everything from being able to touch things, see things, shoot lightning, and pretty much all of chemistry. Meanwhile, the human "story" based Magnatism is about being able to move metals around (because they are "magnetic"), and Electricty magic is about lightning bolts and static electricty.



                                              If you want anthropomorphic magic, magic that makes sense mythologically or in categories humans care about, it has to be powered by an intelligence with a similar mind to ours. That intelligence somehow set up the rules in ways that we find are pleasing.



                                              Spells become "instructions" to this intelligence, who then does what asked. The being being interacted with could be a god-like entity, a demon, or a post-singularity AI, or an ancient forgotten society of super-scientists who left "macros" active that their children used to play with.



                                              In the last case, the affinity of certain people for certain kinds of magic is akin to being able to unlock a certain smart phone with your voice, because it coincidentally is similar to the real owner (it doesn't have to be voice-locked, but you can imagine it being so). The fact that these ancient intelligences let their children play with fireballs would be explained by the fact that everyone in their society was backed up, and repair/resurrection was trivial; that technology doesn't recognize humans as being in their pattern-banks. Humans who can heal are actually using the "veternarian" macros.



                                              Now, once we divorce the physical mechanism from the "kind of magic", the fact that these are "gravity" macros is because they are the kind of thing that the parent programmed for their kid (or maybe bought the "play with gravity" kit for their kid's system).



                                              Your entire environment could be a simulation running in an ancient computer, with the original hosts long exinct. The matter you are walking on and breathing could be programmable computronium (mass-energy optimized to do computation) that emulates being matter so as to leave a "natural" experience for growing up, but being computronium this is just an option; "magic" literally ignores the apparent laws of physics, because the laws of physics as you experience them are just an emulation layer.



                                              Being able to float in either of these cases is little more than a matter of changing the position value of a variable in a specific coordinate space. Now, no human has access to this underlying system; so they cannot insert the wrong arguments and cause the water on the earth to start to orbit the other way around the sun.






                                              share|improve this answer









                                              $endgroup$












                                              • $begingroup$
                                                "There is no spoon"
                                                $endgroup$
                                                – EveryBitHelps
                                                Mar 16 at 7:26















                                              3












                                              $begingroup$

                                              The kind of magic you are talking about is anthropomorphic.




                                              • Slowing down yourself down when falling, so you don't hit the ground at full speed (as near to 0m/s as possible, at the very least you shouldn't be injured by landing) (e.g. jumping from higher up)

                                              • Levitating yourself / other objects, so they don't move in any direction

                                              • Bonus points: still won't move if influenced by another force (e.g. a thrown stone)

                                              • Pushing objects out of your way (e.g. a locked door)

                                              • Ability to combine multiple "gravity fields", so you could simultaneously keep a cup in place and rotate it upwards-down, while still keeping the liquid inside



                                              Gravity is the measurement of the degree to which mass-energy curves space time around it. What you describe above is a bunch of stuff that seems convincing to a human if you described gravity magic to.



                                              A physics based process won't neatly fit in a "stuff that X magic would do" that a human would narrate, because physics doesn't really care what a human finds narratively convincing.



                                              As an example, Electro-Magnitism covers everything from being able to touch things, see things, shoot lightning, and pretty much all of chemistry. Meanwhile, the human "story" based Magnatism is about being able to move metals around (because they are "magnetic"), and Electricty magic is about lightning bolts and static electricty.



                                              If you want anthropomorphic magic, magic that makes sense mythologically or in categories humans care about, it has to be powered by an intelligence with a similar mind to ours. That intelligence somehow set up the rules in ways that we find are pleasing.



                                              Spells become "instructions" to this intelligence, who then does what asked. The being being interacted with could be a god-like entity, a demon, or a post-singularity AI, or an ancient forgotten society of super-scientists who left "macros" active that their children used to play with.



                                              In the last case, the affinity of certain people for certain kinds of magic is akin to being able to unlock a certain smart phone with your voice, because it coincidentally is similar to the real owner (it doesn't have to be voice-locked, but you can imagine it being so). The fact that these ancient intelligences let their children play with fireballs would be explained by the fact that everyone in their society was backed up, and repair/resurrection was trivial; that technology doesn't recognize humans as being in their pattern-banks. Humans who can heal are actually using the "veternarian" macros.



                                              Now, once we divorce the physical mechanism from the "kind of magic", the fact that these are "gravity" macros is because they are the kind of thing that the parent programmed for their kid (or maybe bought the "play with gravity" kit for their kid's system).



                                              Your entire environment could be a simulation running in an ancient computer, with the original hosts long exinct. The matter you are walking on and breathing could be programmable computronium (mass-energy optimized to do computation) that emulates being matter so as to leave a "natural" experience for growing up, but being computronium this is just an option; "magic" literally ignores the apparent laws of physics, because the laws of physics as you experience them are just an emulation layer.



                                              Being able to float in either of these cases is little more than a matter of changing the position value of a variable in a specific coordinate space. Now, no human has access to this underlying system; so they cannot insert the wrong arguments and cause the water on the earth to start to orbit the other way around the sun.






                                              share|improve this answer









                                              $endgroup$












                                              • $begingroup$
                                                "There is no spoon"
                                                $endgroup$
                                                – EveryBitHelps
                                                Mar 16 at 7:26













                                              3












                                              3








                                              3





                                              $begingroup$

                                              The kind of magic you are talking about is anthropomorphic.




                                              • Slowing down yourself down when falling, so you don't hit the ground at full speed (as near to 0m/s as possible, at the very least you shouldn't be injured by landing) (e.g. jumping from higher up)

                                              • Levitating yourself / other objects, so they don't move in any direction

                                              • Bonus points: still won't move if influenced by another force (e.g. a thrown stone)

                                              • Pushing objects out of your way (e.g. a locked door)

                                              • Ability to combine multiple "gravity fields", so you could simultaneously keep a cup in place and rotate it upwards-down, while still keeping the liquid inside



                                              Gravity is the measurement of the degree to which mass-energy curves space time around it. What you describe above is a bunch of stuff that seems convincing to a human if you described gravity magic to.



                                              A physics based process won't neatly fit in a "stuff that X magic would do" that a human would narrate, because physics doesn't really care what a human finds narratively convincing.



                                              As an example, Electro-Magnitism covers everything from being able to touch things, see things, shoot lightning, and pretty much all of chemistry. Meanwhile, the human "story" based Magnatism is about being able to move metals around (because they are "magnetic"), and Electricty magic is about lightning bolts and static electricty.



                                              If you want anthropomorphic magic, magic that makes sense mythologically or in categories humans care about, it has to be powered by an intelligence with a similar mind to ours. That intelligence somehow set up the rules in ways that we find are pleasing.



                                              Spells become "instructions" to this intelligence, who then does what asked. The being being interacted with could be a god-like entity, a demon, or a post-singularity AI, or an ancient forgotten society of super-scientists who left "macros" active that their children used to play with.



                                              In the last case, the affinity of certain people for certain kinds of magic is akin to being able to unlock a certain smart phone with your voice, because it coincidentally is similar to the real owner (it doesn't have to be voice-locked, but you can imagine it being so). The fact that these ancient intelligences let their children play with fireballs would be explained by the fact that everyone in their society was backed up, and repair/resurrection was trivial; that technology doesn't recognize humans as being in their pattern-banks. Humans who can heal are actually using the "veternarian" macros.



                                              Now, once we divorce the physical mechanism from the "kind of magic", the fact that these are "gravity" macros is because they are the kind of thing that the parent programmed for their kid (or maybe bought the "play with gravity" kit for their kid's system).



                                              Your entire environment could be a simulation running in an ancient computer, with the original hosts long exinct. The matter you are walking on and breathing could be programmable computronium (mass-energy optimized to do computation) that emulates being matter so as to leave a "natural" experience for growing up, but being computronium this is just an option; "magic" literally ignores the apparent laws of physics, because the laws of physics as you experience them are just an emulation layer.



                                              Being able to float in either of these cases is little more than a matter of changing the position value of a variable in a specific coordinate space. Now, no human has access to this underlying system; so they cannot insert the wrong arguments and cause the water on the earth to start to orbit the other way around the sun.






                                              share|improve this answer









                                              $endgroup$



                                              The kind of magic you are talking about is anthropomorphic.




                                              • Slowing down yourself down when falling, so you don't hit the ground at full speed (as near to 0m/s as possible, at the very least you shouldn't be injured by landing) (e.g. jumping from higher up)

                                              • Levitating yourself / other objects, so they don't move in any direction

                                              • Bonus points: still won't move if influenced by another force (e.g. a thrown stone)

                                              • Pushing objects out of your way (e.g. a locked door)

                                              • Ability to combine multiple "gravity fields", so you could simultaneously keep a cup in place and rotate it upwards-down, while still keeping the liquid inside



                                              Gravity is the measurement of the degree to which mass-energy curves space time around it. What you describe above is a bunch of stuff that seems convincing to a human if you described gravity magic to.



                                              A physics based process won't neatly fit in a "stuff that X magic would do" that a human would narrate, because physics doesn't really care what a human finds narratively convincing.



                                              As an example, Electro-Magnitism covers everything from being able to touch things, see things, shoot lightning, and pretty much all of chemistry. Meanwhile, the human "story" based Magnatism is about being able to move metals around (because they are "magnetic"), and Electricty magic is about lightning bolts and static electricty.



                                              If you want anthropomorphic magic, magic that makes sense mythologically or in categories humans care about, it has to be powered by an intelligence with a similar mind to ours. That intelligence somehow set up the rules in ways that we find are pleasing.



                                              Spells become "instructions" to this intelligence, who then does what asked. The being being interacted with could be a god-like entity, a demon, or a post-singularity AI, or an ancient forgotten society of super-scientists who left "macros" active that their children used to play with.



                                              In the last case, the affinity of certain people for certain kinds of magic is akin to being able to unlock a certain smart phone with your voice, because it coincidentally is similar to the real owner (it doesn't have to be voice-locked, but you can imagine it being so). The fact that these ancient intelligences let their children play with fireballs would be explained by the fact that everyone in their society was backed up, and repair/resurrection was trivial; that technology doesn't recognize humans as being in their pattern-banks. Humans who can heal are actually using the "veternarian" macros.



                                              Now, once we divorce the physical mechanism from the "kind of magic", the fact that these are "gravity" macros is because they are the kind of thing that the parent programmed for their kid (or maybe bought the "play with gravity" kit for their kid's system).



                                              Your entire environment could be a simulation running in an ancient computer, with the original hosts long exinct. The matter you are walking on and breathing could be programmable computronium (mass-energy optimized to do computation) that emulates being matter so as to leave a "natural" experience for growing up, but being computronium this is just an option; "magic" literally ignores the apparent laws of physics, because the laws of physics as you experience them are just an emulation layer.



                                              Being able to float in either of these cases is little more than a matter of changing the position value of a variable in a specific coordinate space. Now, no human has access to this underlying system; so they cannot insert the wrong arguments and cause the water on the earth to start to orbit the other way around the sun.







                                              share|improve this answer












                                              share|improve this answer



                                              share|improve this answer










                                              answered Mar 15 at 17:43









                                              YakkYakk

                                              9,07111238




                                              9,07111238











                                              • $begingroup$
                                                "There is no spoon"
                                                $endgroup$
                                                – EveryBitHelps
                                                Mar 16 at 7:26
















                                              • $begingroup$
                                                "There is no spoon"
                                                $endgroup$
                                                – EveryBitHelps
                                                Mar 16 at 7:26















                                              $begingroup$
                                              "There is no spoon"
                                              $endgroup$
                                              – EveryBitHelps
                                              Mar 16 at 7:26




                                              $begingroup$
                                              "There is no spoon"
                                              $endgroup$
                                              – EveryBitHelps
                                              Mar 16 at 7:26











                                              3












                                              $begingroup$

                                              I'm borrowing heavily from the "Irregular at the Magic Highschool" light novel series where magic is done by influencing the information bodies of particles. Basically assume everything has an "information body" similar to how in physics simulations every particle has some sort of matrix/vector which stores its properties. These properties can be position, time, velocity, mass, energy, etc... This is somewhat similar to string theory where particles are strings and properties of a particle are determined by the vibration of the string.



                                              Whichever way you want to go about it, gravity magic influences the vibration/information of the particle related to gravity. There are no "gravity fields" in this solution so I can't check off the ability to create multiple gravity fields but you can still rotate a cup and not have any liquid fall out by influencing the particles of the cup along with the liquid inside.






                                              share|improve this answer









                                              $endgroup$

















                                                3












                                                $begingroup$

                                                I'm borrowing heavily from the "Irregular at the Magic Highschool" light novel series where magic is done by influencing the information bodies of particles. Basically assume everything has an "information body" similar to how in physics simulations every particle has some sort of matrix/vector which stores its properties. These properties can be position, time, velocity, mass, energy, etc... This is somewhat similar to string theory where particles are strings and properties of a particle are determined by the vibration of the string.



                                                Whichever way you want to go about it, gravity magic influences the vibration/information of the particle related to gravity. There are no "gravity fields" in this solution so I can't check off the ability to create multiple gravity fields but you can still rotate a cup and not have any liquid fall out by influencing the particles of the cup along with the liquid inside.






                                                share|improve this answer









                                                $endgroup$















                                                  3












                                                  3








                                                  3





                                                  $begingroup$

                                                  I'm borrowing heavily from the "Irregular at the Magic Highschool" light novel series where magic is done by influencing the information bodies of particles. Basically assume everything has an "information body" similar to how in physics simulations every particle has some sort of matrix/vector which stores its properties. These properties can be position, time, velocity, mass, energy, etc... This is somewhat similar to string theory where particles are strings and properties of a particle are determined by the vibration of the string.



                                                  Whichever way you want to go about it, gravity magic influences the vibration/information of the particle related to gravity. There are no "gravity fields" in this solution so I can't check off the ability to create multiple gravity fields but you can still rotate a cup and not have any liquid fall out by influencing the particles of the cup along with the liquid inside.






                                                  share|improve this answer









                                                  $endgroup$



                                                  I'm borrowing heavily from the "Irregular at the Magic Highschool" light novel series where magic is done by influencing the information bodies of particles. Basically assume everything has an "information body" similar to how in physics simulations every particle has some sort of matrix/vector which stores its properties. These properties can be position, time, velocity, mass, energy, etc... This is somewhat similar to string theory where particles are strings and properties of a particle are determined by the vibration of the string.



                                                  Whichever way you want to go about it, gravity magic influences the vibration/information of the particle related to gravity. There are no "gravity fields" in this solution so I can't check off the ability to create multiple gravity fields but you can still rotate a cup and not have any liquid fall out by influencing the particles of the cup along with the liquid inside.







                                                  share|improve this answer












                                                  share|improve this answer



                                                  share|improve this answer










                                                  answered Mar 16 at 3:09









                                                  CptLaskyCptLasky

                                                  514




                                                  514





















                                                      0












                                                      $begingroup$


                                                      This answer will not be scientifically detailed. I will be using simple terms to keep it understandable. Assume some accuracy to be lost as a result.




                                                      "What physical law can magic manipulate in order to affect gravity?"



                                                      Gravity Magic Can Manipulate Gravity.



                                                      What? Were you expecting something different?



                                                      Gravity magic works by localizing a mass of mana in a specific region to cause specific gravitational events in that region.



                                                      What effects does Gravity Magic need to have?



                                                      • Gravity Amplification

                                                      • Gravity Reduction

                                                      • Gravity Nullification

                                                      • Gravity Reversion

                                                      So, how would this work?



                                                      First you need to consider what components influence gravity: Mass and Volume. The greater the mass in a volume of space, the greater the gravity it will influence other objects of mass with.



                                                      Mass is the amount of stuff an object is made of. Volume is the amount of space that stuff fills.



                                                      Next, you need to consider how LONG the increase will occur.



                                                      By increasing the mass of an area, you increase its density. In turn, the amount of gravity will increase. Now, normally, gravitational shifts are minor. If you have a crumpled ball of aluminum foil, that's going to have less gravity than a compressed ball of pure aluminum. By very little, but the gravitational amount will be different. But what about if the amount of mass in that ball was increased by 100 despite maintaining the same volume? Then the gravity would increase a fair amount. Would it be noticeable? Not really because the gravity of the planet would be more influential.



                                                      So, how would we affect the changes you desire?



                                                      • Slowed Descent - When you cast Slowed Descent, the caster casts a moving mass of high-density gravity above them which has its own gravity of 0.9 Gs, bringing the influence of the planet's gravity on you to a 10th of what it was.

                                                      • Levitation - When you cast Levitation, the caster casts a mass in an area which attracts anything nearby to its source. The central source of gravity is surrounded by another layer of gravity which serves to keep objects balanced between the two gravitational sources. In order to quickly drag an object, the gravity would be 5Gs, but would decay over time, quickly reaching 1.1 Gs before it maintains that for a while and decays completely. The higher the gravity and more localized the field, the more difficult it would be for an outside force to move the affected objects. The problem is, the greater the gravity, the more dangerous it is to be inside its direct field of influence.

                                                      • Push - A mass of gravity is created which draws in objects towards it.

                                                      • Combined Fields - Just create two separate masses moving at appropriate speeds in different directions so that they are both affecting the same object and not being negatively influenced each other's gravity.

                                                      The answer is just create masses of mana. They'll emit a gravitational force and that force will grant the desired effects. You could even play with the idea of negative mass, but that would be your own choice of creative liberty.






                                                      share|improve this answer











                                                      $endgroup$

















                                                        0












                                                        $begingroup$


                                                        This answer will not be scientifically detailed. I will be using simple terms to keep it understandable. Assume some accuracy to be lost as a result.




                                                        "What physical law can magic manipulate in order to affect gravity?"



                                                        Gravity Magic Can Manipulate Gravity.



                                                        What? Were you expecting something different?



                                                        Gravity magic works by localizing a mass of mana in a specific region to cause specific gravitational events in that region.



                                                        What effects does Gravity Magic need to have?



                                                        • Gravity Amplification

                                                        • Gravity Reduction

                                                        • Gravity Nullification

                                                        • Gravity Reversion

                                                        So, how would this work?



                                                        First you need to consider what components influence gravity: Mass and Volume. The greater the mass in a volume of space, the greater the gravity it will influence other objects of mass with.



                                                        Mass is the amount of stuff an object is made of. Volume is the amount of space that stuff fills.



                                                        Next, you need to consider how LONG the increase will occur.



                                                        By increasing the mass of an area, you increase its density. In turn, the amount of gravity will increase. Now, normally, gravitational shifts are minor. If you have a crumpled ball of aluminum foil, that's going to have less gravity than a compressed ball of pure aluminum. By very little, but the gravitational amount will be different. But what about if the amount of mass in that ball was increased by 100 despite maintaining the same volume? Then the gravity would increase a fair amount. Would it be noticeable? Not really because the gravity of the planet would be more influential.



                                                        So, how would we affect the changes you desire?



                                                        • Slowed Descent - When you cast Slowed Descent, the caster casts a moving mass of high-density gravity above them which has its own gravity of 0.9 Gs, bringing the influence of the planet's gravity on you to a 10th of what it was.

                                                        • Levitation - When you cast Levitation, the caster casts a mass in an area which attracts anything nearby to its source. The central source of gravity is surrounded by another layer of gravity which serves to keep objects balanced between the two gravitational sources. In order to quickly drag an object, the gravity would be 5Gs, but would decay over time, quickly reaching 1.1 Gs before it maintains that for a while and decays completely. The higher the gravity and more localized the field, the more difficult it would be for an outside force to move the affected objects. The problem is, the greater the gravity, the more dangerous it is to be inside its direct field of influence.

                                                        • Push - A mass of gravity is created which draws in objects towards it.

                                                        • Combined Fields - Just create two separate masses moving at appropriate speeds in different directions so that they are both affecting the same object and not being negatively influenced each other's gravity.

                                                        The answer is just create masses of mana. They'll emit a gravitational force and that force will grant the desired effects. You could even play with the idea of negative mass, but that would be your own choice of creative liberty.






                                                        share|improve this answer











                                                        $endgroup$















                                                          0












                                                          0








                                                          0





                                                          $begingroup$


                                                          This answer will not be scientifically detailed. I will be using simple terms to keep it understandable. Assume some accuracy to be lost as a result.




                                                          "What physical law can magic manipulate in order to affect gravity?"



                                                          Gravity Magic Can Manipulate Gravity.



                                                          What? Were you expecting something different?



                                                          Gravity magic works by localizing a mass of mana in a specific region to cause specific gravitational events in that region.



                                                          What effects does Gravity Magic need to have?



                                                          • Gravity Amplification

                                                          • Gravity Reduction

                                                          • Gravity Nullification

                                                          • Gravity Reversion

                                                          So, how would this work?



                                                          First you need to consider what components influence gravity: Mass and Volume. The greater the mass in a volume of space, the greater the gravity it will influence other objects of mass with.



                                                          Mass is the amount of stuff an object is made of. Volume is the amount of space that stuff fills.



                                                          Next, you need to consider how LONG the increase will occur.



                                                          By increasing the mass of an area, you increase its density. In turn, the amount of gravity will increase. Now, normally, gravitational shifts are minor. If you have a crumpled ball of aluminum foil, that's going to have less gravity than a compressed ball of pure aluminum. By very little, but the gravitational amount will be different. But what about if the amount of mass in that ball was increased by 100 despite maintaining the same volume? Then the gravity would increase a fair amount. Would it be noticeable? Not really because the gravity of the planet would be more influential.



                                                          So, how would we affect the changes you desire?



                                                          • Slowed Descent - When you cast Slowed Descent, the caster casts a moving mass of high-density gravity above them which has its own gravity of 0.9 Gs, bringing the influence of the planet's gravity on you to a 10th of what it was.

                                                          • Levitation - When you cast Levitation, the caster casts a mass in an area which attracts anything nearby to its source. The central source of gravity is surrounded by another layer of gravity which serves to keep objects balanced between the two gravitational sources. In order to quickly drag an object, the gravity would be 5Gs, but would decay over time, quickly reaching 1.1 Gs before it maintains that for a while and decays completely. The higher the gravity and more localized the field, the more difficult it would be for an outside force to move the affected objects. The problem is, the greater the gravity, the more dangerous it is to be inside its direct field of influence.

                                                          • Push - A mass of gravity is created which draws in objects towards it.

                                                          • Combined Fields - Just create two separate masses moving at appropriate speeds in different directions so that they are both affecting the same object and not being negatively influenced each other's gravity.

                                                          The answer is just create masses of mana. They'll emit a gravitational force and that force will grant the desired effects. You could even play with the idea of negative mass, but that would be your own choice of creative liberty.






                                                          share|improve this answer











                                                          $endgroup$




                                                          This answer will not be scientifically detailed. I will be using simple terms to keep it understandable. Assume some accuracy to be lost as a result.




                                                          "What physical law can magic manipulate in order to affect gravity?"



                                                          Gravity Magic Can Manipulate Gravity.



                                                          What? Were you expecting something different?



                                                          Gravity magic works by localizing a mass of mana in a specific region to cause specific gravitational events in that region.



                                                          What effects does Gravity Magic need to have?



                                                          • Gravity Amplification

                                                          • Gravity Reduction

                                                          • Gravity Nullification

                                                          • Gravity Reversion

                                                          So, how would this work?



                                                          First you need to consider what components influence gravity: Mass and Volume. The greater the mass in a volume of space, the greater the gravity it will influence other objects of mass with.



                                                          Mass is the amount of stuff an object is made of. Volume is the amount of space that stuff fills.



                                                          Next, you need to consider how LONG the increase will occur.



                                                          By increasing the mass of an area, you increase its density. In turn, the amount of gravity will increase. Now, normally, gravitational shifts are minor. If you have a crumpled ball of aluminum foil, that's going to have less gravity than a compressed ball of pure aluminum. By very little, but the gravitational amount will be different. But what about if the amount of mass in that ball was increased by 100 despite maintaining the same volume? Then the gravity would increase a fair amount. Would it be noticeable? Not really because the gravity of the planet would be more influential.



                                                          So, how would we affect the changes you desire?



                                                          • Slowed Descent - When you cast Slowed Descent, the caster casts a moving mass of high-density gravity above them which has its own gravity of 0.9 Gs, bringing the influence of the planet's gravity on you to a 10th of what it was.

                                                          • Levitation - When you cast Levitation, the caster casts a mass in an area which attracts anything nearby to its source. The central source of gravity is surrounded by another layer of gravity which serves to keep objects balanced between the two gravitational sources. In order to quickly drag an object, the gravity would be 5Gs, but would decay over time, quickly reaching 1.1 Gs before it maintains that for a while and decays completely. The higher the gravity and more localized the field, the more difficult it would be for an outside force to move the affected objects. The problem is, the greater the gravity, the more dangerous it is to be inside its direct field of influence.

                                                          • Push - A mass of gravity is created which draws in objects towards it.

                                                          • Combined Fields - Just create two separate masses moving at appropriate speeds in different directions so that they are both affecting the same object and not being negatively influenced each other's gravity.

                                                          The answer is just create masses of mana. They'll emit a gravitational force and that force will grant the desired effects. You could even play with the idea of negative mass, but that would be your own choice of creative liberty.







                                                          share|improve this answer














                                                          share|improve this answer



                                                          share|improve this answer








                                                          edited Mar 18 at 3:40

























                                                          answered Mar 18 at 3:32









                                                          Sora TamashiiSora Tamashii

                                                          1,489430




                                                          1,489430



























                                                              draft saved

                                                              draft discarded
















































                                                              Thanks for contributing an answer to Worldbuilding Stack Exchange!


                                                              • Please be sure to answer the question. Provide details and share your research!

                                                              But avoid


                                                              • Asking for help, clarification, or responding to other answers.

                                                              • Making statements based on opinion; back them up with references or personal experience.

                                                              Use MathJax to format equations. MathJax reference.


                                                              To learn more, see our tips on writing great answers.




                                                              draft saved


                                                              draft discarded














                                                              StackExchange.ready(
                                                              function ()
                                                              StackExchange.openid.initPostLogin('.new-post-login', 'https%3a%2f%2fworldbuilding.stackexchange.com%2fquestions%2f141533%2fgravity-magic-how-does-it-work%23new-answer', 'question_page');

                                                              );

                                                              Post as a guest















                                                              Required, but never shown





















































                                                              Required, but never shown














                                                              Required, but never shown












                                                              Required, but never shown







                                                              Required, but never shown

































                                                              Required, but never shown














                                                              Required, but never shown












                                                              Required, but never shown







                                                              Required, but never shown






                                                              Popular posts from this blog

                                                              How to check contact read email or not when send email to Individual?

                                                              Bahrain

                                                              Postfix configuration issue with fips on centos 7; mailgun relay